Középiskolai Matematikai és Fizikai Lapok
Informatika rovattal
Kiadja a MATFUND Alapítvány
Már regisztráltál?
Új vendég vagy?

Fórum: A Goldbach-sejtésről

  [1]    [2]    [3]    [4]    [5]    [6]    [7]    [8]    [9]    [10]    [11]    [12]    [13]    [14]    [15]    [16]    [17]    [18]    [19]    [20]    [21]    [22]    [23]    [24]    [25]    [26]    [27]    [28]    [29]    [30]    [31]    [32]    [33]    [34]    [35]    [36]    [37]    [38]    [39]    [40]    [41]    [42]    [43]    [44]    [45]    [46]    [47]    [48]    [49]    [50]  

Szeretnél hozzászólni? Jelentkezz be.
[658] bily712010-05-09 08:03:10

(b)

(i) Végtelen sok Fermat-szám van.

(ii)Bármely két Fermat-szám relatív prím.

(iii)Mindegyiknek Fermat-számnak van 8k+1 alakú prímosztója, amely prímosztók egyben 4(2k)+1 alakúak.

(i)-ból következik, hogy végtelen sok ilyen prímosztó van, (ii)-ből pedig, hogy ezek páronként különböznek, ergo végtelen sok 4k+1 alakú prím van.

Az ikerprím'-ből meg úgy következik, hogy ha végtelen sok ikerprím van, akkor végtelen sok olyan számpárunk van, ahol az egyik prím, 4k-1, a másik 4k+1 alakú.

Előzmény: [655] bily71, 2010-05-08 22:06:23
[657] Róbert Gida2010-05-08 22:38:30

5 pontos feladatnál talán adnék 1 pontot a *bizonyításodra*.

Előzmény: [655] bily71, 2010-05-08 22:06:23
[656] Róbert Gida2010-05-08 22:32:30

"gondolkodunk" Legfeljebb te.

Előzmény: [654] Zilberbach, 2010-05-08 21:56:18
[655] bily712010-05-08 22:06:23

(a) Végtelen sok 4a-1 alakú szám van, mindegyiknek van 4k-1 alakú osztója, ha véges sok ilyen alakú prím van és ezeket összeszorozva kapjuk 4a-t, akkor 4a-1-nek van ezektől a prímektől különböző prímosztója. Tehát az a feltételezésünk, hogy véges ok ilyen prím van ellentmondás.

Előzmény: [648] Fálesz Mihály, 2010-05-08 20:48:48
[654] Zilberbach2010-05-08 21:56:18

Ha a természetes számok körében gondolkodunk, akkor nincs igazad, mert a -1 már ezen kívül esik.

Előzmény: [653] Róbert Gida, 2010-05-08 21:35:57
[653] Róbert Gida2010-05-08 21:35:57

bily-vel nem vagytok testvérek? Amit írsz nagyon zöld, és persze semmit nem bizonyítasz.

"3. Négynél kisebb páratlan szám viszont csak kettő van; az egy és a három."

Négynél kisebb a -1 és páratlan. Na ennyit a hasonló postok színvonaláról.

Előzmény: [651] Zilberbach, 2010-05-08 21:03:03
[652] Róbert Gida2010-05-08 21:27:27

"Ezt nehezen hiszem el, tájékozódtam felőled a honlapodon, (ami nyílvános), és tudom, hogy ennél sokkal bonyolultabb dolgokat is átlátsz."

Még szép, Péter végzett matematikus.

Ma kb. 10 percet rászántam a hozzászólásaidra. Már ahogy kezded az állításaidat az rossz, kéne írni, hogy minden x-re, vagy hasonló... Bizonyításaidat meg sehol nem fogadnák el. Mi az, hogy egy szám aszimptotája egy másik számnak? Ez így zöldség.

627-es hozzászólásodban f1(x) valójában a 2,3,5 egyikével sem osztható pozitiv egész számok száma x-ig (ha x>0). Ez persze nemnegatív, így f1(x)\ge0 triviálisan. És persze semmi köze nincs a Goldbach sejtéshez, se az ikerprímekhez.

Előzmény: [649] bily71, 2010-05-08 20:52:00
[651] Zilberbach2010-05-08 21:03:03

1. A maradéknak, ha néggyel osztunk kisebbnek kell lennie mint négy.

2. Ezenfelül a maradéknak páratlannak kell lennie, ha 2-nél nagyobb prímszámot osztunk mert az elosztott prímszám páratlan, és mi egy páros számot, a négyet vonogatjuk ki belőle - ahányszor csak lehet - az osztás során.

3. Négynél kisebb páratlan szám viszont csak kettő van; az egy és a három.

4. Ha a maradék egy, akkor a prím egyel nagyobb, mint a négy többszöröse.

5. Ha a maradék három, akor a prím eggyel kisebb mint a négy (következő) többszöröse.

Előzmény: [640] Maga Péter, 2010-05-08 10:06:03
[650] Róbert Gida2010-05-08 20:53:17

Nem viccel, és köze nincs a postodnak a 640-es hozzászóláshoz.

Előzmény: [646] bily71, 2010-05-08 19:58:59
[649] bily712010-05-08 20:52:00

Péter!

Lenne egy kérdésem. Amit [603]-tól írtam, azt Te sem érted? Ezt nehezen hiszem el, tájékozódtam felőled a honlapodon, (ami nyílvános), és tudom, hogy ennél sokkal bonyolultabb dolgokat is átlátsz.

Legutóbb addig jutottunk, hogy az ikerprím-sejtés ekvivalens azzal, (egy sor ekvivalens átfogalmazás után), hogy bizonyos számtani sorozatokban hány darab egész nem szerepel. Ezeket a számokat megszámoltuk x-ig és kaptunk egy függvényt, amit összehasonlítottam azzal a függvénnyel, ahol elhagytuk az egészrészjeleket. Erre azt mondtad, hogy a becslésem nem biztos, hogy helytálló. Ennek eldöntésére [603]-tól felvázoltam egy viszonylag egyszerű eljárást, ami úgy látszik működik. Miért nem jó, ha egyszer működik? Csupán csak azért, mert én írtam?

Előzmény: [640] Maga Péter, 2010-05-08 10:06:03
[648] Fálesz Mihály2010-05-08 20:48:48

Szerintem nem viccel. Két különálló feladatról van szó.

(a) Végtelen sok 4k-1 alakú prím van.

(b) Végtelen sok 4k+1 alakú prím van.

Előzmény: [646] bily71, 2010-05-08 19:58:59
[647] bily712010-05-08 20:35:10

Még annyit, hogy végtelen sok 4k-1 alakú páronként relatív prím összetett létezik, ezek bonthatók így: 4k-1=(4a-1)(4b+1), és mivel relativ prímek, a két tényező felbontása páronként különböző prímekbők áll, és ha 4a-1 összetett, tovább bomlik így: 4a-1=(4c-1)(4d+1), ahol a két tényező felbontása páronként különböző prímekből áll, és ha..., vagyis a végén biztos kapunk két prím tényezőt, amelyek legyenek p=4u-1 és q=4v+1, és az első mondatból következik, hogy ilyen párból végtelen sok páronként különböző van.

Előzmény: [646] bily71, 2010-05-08 19:58:59
[646] bily712010-05-08 19:58:59

Most viccelsz?

Ha p\equiv0 (mod 4), ekkor 4|p, ilyen prím nincs.

Ha p\equiv2 (mod 4), akkor 2|p, ilyen prím csak egy van, a kettő.

Az összes többi prím páratlan, vagyis, ha p prím és 2 ³ p, akkor p\equiv\pm1 (mod 4), páratlan prímekből pedig végtelen sok van...

Előzmény: [640] Maga Péter, 2010-05-08 10:06:03
[645] jenei.attila2010-05-08 14:54:04

Az valóban nem az. De hát mégiscsak kell hozzá egy kis matek.

Előzmény: [644] Zilberbach, 2010-05-08 13:54:01
[644] Zilberbach2010-05-08 13:54:01

Az orvostudomány egy olyan ága, ahol a gyógyító tevékenység végzéséhez elég annyi matematika tudás is, mint amivel én rendelkezem. Biztosítalak róla, hogy nem áltudomány!

Előzmény: [643] jenei.attila, 2010-05-08 13:41:35
[643] jenei.attila2010-05-08 13:41:35

Meg szabad kérdezni, hogy mi az a tudományterület amiben "vérprofi szinten ki vagy képezve"? Mert ha az valóban tudomány (és nem áltudomány), akkor csak van valami köze a matematikához.

Előzmény: [642] Zilberbach, 2010-05-08 12:02:55
[642] Zilberbach2010-05-08 12:02:55

Kedves bily!

Szerintem gondold át Tóbi és más fórumozók neked szánt tanácsait.

Írtál arról hogy komoly anyagi problémáid vannak, és ezekből a fórumon végzet tevékenységeddel szeretnél kitörni. No erre is ráillik szerintem Fernandó megjegyzése az elenyészően csekély valószínűségről.

Ne rólam végy példát, mert én a tiednél jóval szerényebb matek tudással is nyugodtan fórumozhatok. Egy matematikától távoli tudományterületen vérprofi szinten ki vagyok képezve, és ebből elfogadhatóan meg is tudok élni a családommal együtt.

[641] Zilberbach2010-05-08 11:49:36

Kedves Maga Péter!

Sajnos most így kapásból nem tudom bizonyítani - sőt az is lehet hogy soha nem jutok el arra a szintre - de azért majd gondolkodom rajta, hátha menni fog.

Előzmény: [640] Maga Péter, 2010-05-08 10:06:03
[640] Maga Péter2010-05-08 10:06:03

Bily, Zilberbach, azt be tudjátok bizonyítani, hogy végtelen sok 4k-1 és végtelen sok 4k+1 alakú prím van? Csak azért kérdezem, mert akár a Goldbach'-ból, akár az ikerprím'-ből könnyen következik.

[639] Maga Péter2010-05-08 10:04:26

Ez olyan naiv, hogy nem is igaz... értem én, mit értesz azon, hogy ,,páronként azonosak'', de azt nem így mondjuk.

Előzmény: [636] Zilberbach, 2010-05-08 08:40:41
[638] Maga Péter2010-05-08 09:53:58

Mosom kezeimet: az [561]-es hozzászólásomban megpróbáltam matematizálni a témát, de ezzel csúnyán fölé lőttem. A fórum bezárásával kapcsolatban két problémát látok. Egyrészt voltak itt értelmes hozzászólások, nem lenne jogos azokat is eltüntetni a sok ocsúval együtt, ami körülveszi őket. Másrészt a téma nincs eleve alacsony színvonalra rendelve; attól, hogy nem középiskolás szintűek az itt vitatott problémák, lehet róluk értelmesen beszélgetni egy középiskolás fórumon is. Ennek természetesen előfeltétele az a hozzáállás, hogy nem csak a problémák megoldását tartjuk eredménynek.

Előzmény: [632] Tóbi, 2010-05-07 23:00:31
[637] Zilberbach2010-05-08 09:08:25

Egyetértek bily és Fernando legutóbbi hozzászólásaival. Én sokat tanultam a fórumon - miközben sajnos alaposan levittem a színvonalat - de lehet hogy a newtoni hatás-ellenhatás törvénye néha a matematikai fórumokon is érvényesül.

[636] Zilberbach2010-05-08 08:40:41

Mit szóltok az alábbi naiv ötlethez: Bizonyítandó lenne az a sejtés, miszerint minden 10-nél nagyobb páros szám előállítható 4 db. olyan prím összegként, amelyek páronként azonosak.

[635] bily712010-05-08 08:26:21

A matematikában, és minden tudományban elfogadhatatlan a bírálat, ha az a személyre irányul és nem a gondolatmenetre. Nem kötelező megvizsgálni minden firkálmányt, de a megértés, vagy elolvasás nélküli véleményezés nem tudományos cselekedet, csak személyeskedés.

Egy internetes fórum, mégha tudományos is, elsősorban a kellemes csevegést kell, hogy szolgálja, persze itt matemtikai témájú csevegést. Miért ne beszélgethetnénk ezekről a sejtésekről, mi, buta "diákok"? Szerintem kár lerohanni minden középiskolást, ha eszébe jut valami. Ez most nem egy vizsga. Bizonyos diktatúrákban még az ovodások sem beszélgethetnek akármiről, de az ilyen nevelés is ugyanolyan káros hatással van, mintha nem felügyelnénk őket, megdorgálva a csúnyán beszélőket. Azt is furcsának találom, hogy sok forumozó valami feléje irányuló személyes sértésként kezel egy-két tévedést.

Találtatok hibát? Hol van? Ha megmutatjátok hol a hiba, abból mindenki csak tanulhat. Persze a jó gondolatmeneteket kell megértetni egy diákkal, de néha azt is meg lehet mutatni, hogyan eshet csapdába, és ilyenkor bizony rossz útvonalon kell végigvezetni.

Tehát, ez a gondolatom támadt, ha akarjátok gondoljátok át ti is. A zárójelezéssel lépésről-lépésre mindig olyan függvényt kapunk, amelynek "aszimptotája" párhuzamos azzal a függvénnyel, amelyet úgy kapunk, hogy elhagyjuk az egészrészjeleket és felette halad, vagy a távolságuk nulla, vagyis azonosak. Soha nem kapunk olyan függvényt, amelynek "aszimptotája" az egészrészjelek nélküli függvény alatt haladna. Ez azt jelenti, hogy az ilyen függvények nem lehetnek korlátosak. Vagy nem jól következtettem?

Előzmény: [631] bily71, 2010-05-07 22:42:09
[634] Fernando2010-05-07 23:36:31

Kedves Tóbi!

Már a topic címe is sokat elárul... Ha komolyra veszem a figurát, akkor egyet kell hogy értsek veled, ha itt az a cél, hogy ezeket a problémákat bizonyítsák. Nem mindig jó komolyra venni a figurát persze. A topic bezáratását még komoly állapotomban is túlzásnak érzem. A KöMaL középiskolásoknak szánt és nagyon értékes lap, a fórumában azonban többen is messze a középiskolás apparátus fölöttivel rendelkeznek (távol álljon tőlem lebecsülni a tehetséges középiskolásokat), sőt nagyon komoly tudással felvértezett emberek is vannak itt, tiszteletem azoknak, akiknek már van mire szerénynek lenniök. Ebben a topicban a matematika mennyisége valóban egyre csökken, de azt nem hinném, hogy ártalmas lenne, néha talán tanulságos, néha szórakoztató. A topic bezáratása esetén lenne legalább még egy topic bezáratási javaslatom :)

Üdv.: Feri

Előzmény: [632] Tóbi, 2010-05-07 23:00:31

  [1]    [2]    [3]    [4]    [5]    [6]    [7]    [8]    [9]    [10]    [11]    [12]    [13]    [14]    [15]    [16]    [17]    [18]    [19]    [20]    [21]    [22]    [23]    [24]    [25]    [26]    [27]    [28]    [29]    [30]    [31]    [32]    [33]    [34]    [35]    [36]    [37]    [38]    [39]    [40]    [41]    [42]    [43]    [44]    [45]    [46]    [47]    [48]    [49]    [50]